Cuantizar un campo complejo de Klein-Gordon: ¿Por qué hay dos tipos de excitaciones?

En la mayoría de las referencias que he visto (ver, por ejemplo, el problema 2.2 de Peskin y Schroeder, o la sección 2.5 aquí ), uno construye el operador de campo ϕ ^ ϕ ^ para el complejo campo Klein-Gordon como sigue:

Primero, tomas la densidad Lagrangiana para el campo clásico de Klein-Gordon

L = μ ϕ μ ϕ - m 2 ϕ ϕ (1) (1) L = μ ϕ μ ϕ - metro 2 ϕ ϕ
y encontrar el impulso conjugado al campo. ϕ ϕ vía

π = L ϕ ˙ = ϕ ˙ . (2) (2) π = L ϕ ˙ = ϕ ˙ .
Entonces, uno impone las relaciones de conmutación canónicas habituales sobre ϕ ^ ϕ ^ y π ^ π ^ :

[ ϕ ^ ( x ) , π ^ ( y ) ] = i δ 3 ( x - y ) . (3) (3) El ϕ ^ ( X ) , π ^ ( y ) ] = yo δ 3 ( X - y ) .
Entonces, uno necesita encontrar operadores ϕ ^ ϕ ^ y π ^ π ^ tal que obedecen las relaciones de conmutación anteriores, y tal que π ^ = ϕ ˙ π ^ = ϕ ˙ . Los libros de texto luego continúan para mostrar que la definición

ϕ ^ ( x ) = re 3 pag ( 2 π ) 3 1 2 p 0 - - - [ a pag mi - i p μ X μ + b pag mi i p μ X μ ] (4) (4) ϕ ^ ( X ) = re 3 pag ( 2 π ) 3 1 2 pag 0 El una pag mi - yo pag μ X μ + segundo pag mi yo pag μ X μ ]
π ^ ( x ) = i re 3 pag ( 2 π ) 3 pag 0 2 - - - [ a pag mi i p μ X μ - b pag mi - i p μ X μ ] (5) (5) π ^ ( X ) = yo re 3 pag ( 2 π ) 3 pag 0 2 El una pag mi yo pag μ X μ - segundo pag mi - yo pag μ X μ ]
dónde una una y segundo segundo Son operadores de aniquilación bosónica, satisfacen estas propiedades.

Mi pregunta es: ¿Por qué necesitamos dos operadores de partículas diferentes para definir ϕ ^ ϕ ^ y π ^ π ^ ? Me parece que uno podría simplemente definir

ϕ ^ ( x ) = re 3 pag ( 2 π ) 3 1 2 p 0 - - - una pag mi - i p μ X μ (6) (6) ϕ ^ ( X ) = re 3 pag ( 2 π ) 3 1 2 pag 0 una pag mi - yo pag μ X μ
π ^ ( x ) = i re 3 pag ( 2 π ) 3 pag 0 2 - - - una pag mi i p μ X μ (7) (7) π ^ ( X ) = yo re 3 pag ( 2 π ) 3 pag 0 2 una pag mi yo pag μ X μ
con una ^ pag una ^ pag Un solo operador de aniquilación bosónica. Entonces claramente π ^ = ϕ ^ ˙ π ^ = ϕ ^ ˙ , y también

[ ϕ ^ ( x ) , π ^ ( y ) ] = = = = yo re 3 pag ( 2 π ) 3 re 3 q ( 2 π ) 3 q 0 4 p 0 - - - mi yo ( q μ y μ - p μ X μ ) [ a pag un q ] yo re 3 pag ( 2 π ) 3 re 3 q ( 2 π ) 3 q 0 4 p 0 - - - mi yo ( q μ y μ - p μ X μ ) ( 2 π ) 3 δ 3 ( p - q ) yo re 3 pag ( 2 π ) 3 1 2 mi i p μ ( y μ - x μ ) yo 2 δ 3 ( y - x ) (8) (8) El ϕ ^ ( X ) , π ^ ( y ) ] = yo re 3 pag ( 2 π ) 3 re 3 q ( 2 π ) 3 q 0 4 pag 0 mi yo ( q μ y μ - pag μ X μ ) El una pag , una q ] = yo re 3 pag ( 2 π ) 3 re 3 q ( 2 π ) 3 q 0 4 pag 0 mi yo ( q μ y μ - pag μ X μ ) ( 2 π ) 3 δ 3 ( pag - q ) = yo re 3 pag ( 2 π ) 3 1 2 mi yo pag μ ( y μ - X μ ) = yo 2 δ 3 ( y - X )

que es, hasta algunos detalles sobre la normalización de la una ^ pag una ^ pag correcto Entonces tendríamos un campo de Klein-Gordon con un solo tipo de excitación, el una ^ pag una ^ pag excitación. ¿Por qué todos los libros de texto afirman que necesitamos dos excitaciones bosónicas separadas, una ^ pag una ^ pag y segundo ^ pag segundo ^ pag ?

En resumen, el libro de texto está engañando. El simple hecho de encontrar un par de operadores que satisfagan la relación de conmutación no es suficiente, también debe utilizar todos los grados de libertad en el campo. De lo contrario, con lo que termine será solo una descripción incompleta del comportamiento del campo, por ejemplo, omitirá algunos de los tipos de partículas. :-) (Ahora, exactamente cómo sabes cuándo has agotado todos los grados de libertad que no tengo idea).
En resumen, el ansatz (6) de OP carece de los modos de frecuencia / energía negativos.

Respuestas (5)

El punto es que el procedimiento de cuantificación generalmente solo es válido para observables físicos de valor real. Todas las versiones de tratar los observables clásicos como funciones reales en el espacio de fase (las cosas se complican más para los fermiones, que ignoraré en este tema), y asociar los observables cuánticos a esos. Por ejemplo, el operador de aniquilación de oscilador armónico a = x + i p una = X + yo pag No es realmente un objeto al que se le permite mirar en la mecánica hamiltoniana clásica: no se producen funciones complejas de valor, o más bien, no son diferentes de solo un par de funciones de valor real que representan la parte real e imaginaria.

Por lo tanto, para cuantificar un campo escalar complejo. ϕ ϕ , hay que escribirlo como ϕ = ϕ 1 ( x ) + i ϕ 2 ( x ) ϕ = ϕ 1 ( X ) + yo ϕ 2 ( X ) , y cuantificar ambos campos escalares reales por separado. Esto produce la expansión de modo habitual del campo escalar complejo con dos conjuntos diferentes de operadores de creación / aniquilación. Para un campo real, podemos tratar una pag una pag y una pag una pag Como operadores porque pueden obtenerlos a partir de la transformada de Fourier de los campos. ϕ ( x ) ϕ ( X ) y π ( x ) π ( X ) , que son valores reales y por lo tanto operadores después de la cuantización. Tanto la transformada de Fourier como el cómputo de una pag una pag y una pag una pag debe pensarse que se lleva a cabo después de la cuantificación para ser consistente con la derivación de las relaciones de conmutación de una pag un pag una pag , una pag de la CCR de ϕ ϕ y π π .

Además, tenga en cuenta que su intento es inconsistente con la cuantización del campo escalar real de otra manera: cuando imponemos ϕ = ϕ ϕ = ϕ en su campo escalar, también obtenemos a = a una = una porque ϕ ˙ = ϕ ˙ = π ϕ ˙ = ϕ ˙ = π en ese caso, lo que contradice su relación de conmutación no nula. Por lo tanto, su versión de la cuantización del campo escalar complejo no se reduce a la cuantificación del campo escalar real y, por lo tanto, es una prescripción de cuantificación completamente diferente.

Entonces, ¿por qué obtenemos un solo operador de creación al realizar el mismo procedimiento con la ecuación de Schrodinger para obtener un campo de Schrodinger? Allí, tenemos una función de onda de valor complejo (pensada como un campo clásico), pero un solo operador de creación es suficiente para determinar el operador de campo.
@JahanClaes No estoy seguro de qué es un campo de Schrödinger, pero estoy seguro de que lo que sea que comience no es un sistema hamiltoniano clásico, por lo que su "cuantización" es totalmente ad hoc y no es un ejemplo de cuantización canónica.
Un campo de Schrodinger es un campo clásico que obedece a la ecuación de Schrodinger. Tiene hamiltoniano H = π ( 1 - d re t - yo 2 ) ψ H = π ( 1 - re re t - yo 2 ) ψ , que supongo que debería ser visto como dos campos reales acoplados?
@JahanClaes Ese no es el hamiltoniano correcto, los hamiltonianos no contienen derivados del tiempo. En cualquier caso, el impulso canónico correcto de un campo de Schrödinger es justo π = i ψ π = yo ψ asi que π π y ψ ψ no son independientes y tiene un sistema Hamiltoniano restringido y la cuantificación formal requeriría aplicar la receta de Dirac-Bergmann, durante la cual se eliminan algunas variables. Ese Wiki parece tomar el atajo aquí que simplemente puedes tomar ψ ψ y ψ ψ Como sus variables es probablemente el resultado final, pero no a priori justificado.
¿Cómo podría el hamiltoniano no contener un derivado del tiempo? El impulso canónico, como usted señaló, no tiene una derivada del tiempo, por lo que no puede eliminar la derivada de ψ ψ a favor de π π . ¿Quiere decir que acaba de mover el tiempo derivado a la π π ¿A través de la integración por partes?

En primer lugar y personalmente, no me gustan los primeros capítulos de Peskin y Schroeder. Creo que es un libro mejor para los profesores que para los estudiantes que aprenden la materia por primera vez. (Otoh creo que se pone mejor después).

Creo que es más instructivo seguir a Srednicki, por ejemplo, en este caso. También utiliza Srednicki. d i a g ( - 1 , 1 , 1 , 1 ) re yo una sol ( - 1 , 1 , 1 , 1 ) convención métrica, pero en algunos casos puede alternar entre las convenciones métricas insertando - 1 - 1 y yo yo 's.

Primero tomamos un campo escalar clásico con la intención de la cuantización canónica a la Dirac.

Escribimos el Lagrangiano para un campo escalar de valor complejo libre:

L = - 1 2 μ ϕ μ ϕ - 1 2 metro 2 | ϕ | 2 + Ω 0 L = - 1 2 μ ϕ μ ϕ - 1 2 metro 2 | ϕ | 2 + Ω 0

Ya que en este momento ϕ ϕ Es solo una función de valor complejo, es decir, ϕ ( x ) ϕ ( X ) es un numero, tenemos ϕ = ϕ ϕ = ϕ , por lo que igualmente podría escribir el Lagrangiano con dagas.

Tomamos nota de la ecuación de movimiento para ϕ ϕ es:

( - + m 2 ) ϕ = 0 ( - + metro 2 ) ϕ = 0

Ahora algunas funciones que satisfacen este pde son

exp ( i k x ± i ω t ) exp ( yo k X ± yo ω t )

dónde k k es un vector de onda real arbitrario, y ω ω es:

ω = + k 2 + m 2 - - - - - - - ω = + k 2 + metro 2
Uno puede imaginar expandir una solución. ϕ ϕ en términos de estas ondas planas, como lo harías al resolver otras pdes. Así escribimos (incluso en el caso de que ϕ ϕ es valorado real) ϕ ϕ como:

ϕ ( x , t ) = re 3 k F ( k ) [ a ( k ) e i k x - i ω t + b ( k ) e i k x + i ω t ] ϕ ( X , t ) = re 3 k F ( k ) El una ( k ) mi yo k X - yo ω t + segundo ( k ) mi yo k X + yo ω t ]

con coeficientes (todavia no operadores) a ( k ) una ( k ) y b ( k ) segundo ( k ) , ya que aún no hay razón para asumir que deberían estar relacionados. F ( k = | k | ) F ( k = | k | ) se inserta por la razón posterior de hacer que la medida de integración lorentz sea invariante y será proporcional a ω ω .

Si ϕ ϕ era real ϕ = ϕ ϕ = ϕ , entonces tendríamos:

ϕ ( x , t ) ϕ ( x , t ) = re 3 k F ( k ) [ a ( k ) e - i k x + i ω t + b ( k ) e - i k x - i ω t ] = re 3 k F ( k ) [ a ( - k ) e + i k x + i ω t + b ( - k ) e + i k x - i ω t ] = re 3 k F ( k ) [ a ( k ) e i k x - i ω t + b ( k ) e i k x + i ω t ] ϕ ( X , t ) = re 3 k F ( k ) El una ( k ) mi - yo k X + yo ω t + segundo ( k ) mi - yo k X - yo ω t ] = re 3 k F ( k ) El una ( - k ) mi + yo k X + yo ω t + segundo ( - k ) mi + yo k X - yo ω t ] ϕ ( X , t ) = re 3 k F ( k ) El una ( k ) mi yo k X - yo ω t + segundo ( k ) mi yo k X + yo ω t ]

Simplemente cambiamos k - k k - k en línea 2 2 . Comparando encontramos que a ( k ) = b ( - k ) una ( k ) = segundo ( - k ) o una ( - k ) = b ( k ) una ( - k ) = segundo ( k ) . Subestando en esto por ϕ ϕ ,

ϕ ( x , t ) = re 3 k F ( k ) [ a ( k ) e i k x - i ω t + a ( - k ) e i k x + i ω t ] = re 3 k F ( k ) [ a ( k ) e i k x - i ω t + a ( k ) e - i k x + i ω t ] = re 3 k F ( k ) [ a ( k ) e i k μ X μ + a ( k ) e - i k μ X μ ] ϕ ( X , t ) = re 3 k F ( k ) El una ( k ) mi yo k X - yo ω t + una ( - k ) mi yo k X + yo ω t ] = re 3 k F ( k ) El una ( k ) mi yo k X - yo ω t + una ( k ) mi - yo k X + yo ω t ] = re 3 k F ( k ) El una ( k ) mi yo k μ X μ + una ( k ) mi - yo k μ X μ ]

Para reconciliarte con P&S puedes cambiar. k p k pag con un 1 1 En unidades naturales. También puede calcular π π Por el impulso conjugado y diferenciador. Puedes cuantificar canónicamente ϕ ϕ y π π después o elegir cuantizar una una 's. Cambio una una a una una etc.

De todos modos, algunas diferencias entre el campo escalar real y complejo serán que en el campo real obtienes partículas neutras, y el campo complejo obtienes dos partículas cargadas, de cargas opuestas. Esto viene de la simetría de la fase de selección del campo. ϕ ϕ en el lagrangiano, que no tienes para el campo escalar real. También hay que hacer una reinterpretación de la segundo segundo operador en el caso complejo para evitar tener estados de energía negativos, el problema del mar de Dirac esencialmente. Asi que segundo segundo Debería acabar creando antipartículas. Verás algo de este tipo con la cuantización del campo de Dirac de todos modos.

Esto es bueno, pero no explica por qué una ^ una ^ Debería ser un operador de aniquilación bosónica cuando cuantifique la teoría. Aún necesita seguir el procedimiento de cuantización canónica para obtener relaciones de conmutación, ¿no?
@JahanClaes Entonces eso es otra cosa que Srednicki muestra. Tú eliges cuantizar canónicamente ϕ ϕ y π π como lo haría para la posición y el impulso, excepto con la función delta de Dirac. O simplemente puede comenzar con el Hamiltoniano que tendría en términos de una una y cuantifican canonicamente la una una 's. Pero tiene que ser Bosonic para spin cero, y en general spin entero. De lo contrario, te enfrentas a todo tipo de problemas, como que el hamiltoniano sea constante y que las cosas no se desvanezcan fuera del lightcone.
@JahanClaes También puedes leer las secciones por ti mismo en línea. En la página que vinculé, el autor hace un borrador disponible en línea, que estaría más cerca de la primera edición impresa. En ese sentido, también tendría errores tipográficos similares, no creo que sean demasiado importantes en los primeros capítulos. Están listados en la página web de todos modos.

Si ϕ ϕ solo tiene un oscilador independiente en la descomposición de Fourier, no es la solución más general de las ecuaciones de Euler-Lagrange (de eom). El campo ϕ ϕ Es decir, antes de la cuantificación, solo un número complejo y, por lo tanto, debe tener 2 grados de libertad independientes, no uno como se escribe arriba.

Pero cuando hacemos lo mismo con la ecuación de Schrodinger para obtener un campo de Schrodinger, solo se necesita un operador de creación / aniquilación, aunque el campo de Schrodinger tenga un valor complejo.
La ecuación de Schrödinger es diferente. Poniendo
Por el amor de Dios. Hice un lío. ¿Por qué no puedo borrar mi comentario? La ecuación de Schrödinger no permite osciladores. La identidad
yo Ψ ˙ = E Ψ yo Ψ ˙ = mi Ψ
solo ponlos a una constante. Por ejemplo, eche un vistazo a la partícula libre en es.wikipedia.org/wiki/Schr%C3%B6dinger_equation (también debe agregar la parte dependiente del tiempo allí).

Existe una manera conceptualmente simple (pero complicada) de relacionar esta expansión con la expansión de Fourier habitual. TL; DR: Requerir ϕ ϕ para satisfacer la ecuación de Klein-Gordon, se dividen los componentes de Fourier distintos de cero en dos clases, correspondientes a partículas y antipartículas.

Para un campo escalar complejo general definido en espacio-tiempo,

ϕ ( x ) = re 4 pag ( 2 π ) 4 ϕ ^ ( p ) e - i p x . (1) (1) ϕ ( X ) = re 4 pag ( 2 π ) 4 ϕ ^ ( pag ) mi - yo pag X .
Esta es la expansión de Fourier ordinaria de cuatro dimensiones . Ahora, impongamos la ecuación de Klein-Gordon
0 = ( 2 + m 2 ) ϕ = re 4 pag ( 2 π ) 4 ( - p 2 + m 2 ) ϕ ^ ( p ) e - i p x . 0 = ( 2 + metro 2 ) ϕ = re 4 pag ( 2 π ) 4 ( - pag 2 + metro 2 ) ϕ ^ ( pag ) mi - yo pag X .
Un campo general que satisfaga esto debe consistir únicamente en modos donde pag 2 = m 2 pag 2 = metro 2 , es decir , los modos en shell . Así,
ϕ ^ ( p ) = 2 π δ ( p 2 - m 2 ) f ( p ) ϕ ^ ( pag ) = 2 π δ ( pag 2 - metro 2 ) F ( pag )
para alguna función F F (el factor 2 π 2 π es conveniente para comparar con la expansión estándar). Escritura p = ( p 0 , p ) pag = ( pag 0 , pag ) ,
δ ( p 2 - m 2 ) = δ ( ( p 0 ) 2 - ( p 2 + m 2 ) ) . δ ( pag 2 - metro 2 ) = δ ( ( pag 0 ) 2 - ( pag 2 + metro 2 ) ) .
El argumento de la δ δ -función tiene dos ceros pag 0 = ± p 2 + m 2 - - - - - - - pag 0 = ± pag 2 + metro 2 (para fijo pag pag ), entonces usamos la regla general
δ ( f ( x ) ) = F ( x yo ) = 0 1 | F ( x yo ) | δ ( x - x yo ) δ ( F ( X ) ) = F ( X yo ) = 0 1 | F ( X yo ) | δ ( X - X yo )
encontrar
δ ( p 2 - m 2 ) = 1 | 2 p 0 | [ δ ( p 0 - p 2 + m 2 - - - - - - - ) + δ ( p 0 + p 2 + m 2 - - - - - - - ) ] . δ ( pag 2 - metro 2 ) = 1 | 2 pag 0 | El δ ( pag 0 - pag 2 + metro 2 ) + δ ( pag 0 + pag 2 + metro 2 ) ] .
Poner esto de nuevo en 1 1 ) y realizando el pag 0 pag 0 integral (abreviando pag 2 + m 2 - - - - - - - = E pag pag 2 + metro 2 = mi pag ):
ϕ ( x ) = re 3 pag ( 2 π ) 3 1 2 E pag [ f ( E pag , p ) e - yo ( E pag , p ) x + f ( - E pag , p ) e - i ( - E pag , p ) x ] = re 3 pag ( 2 π ) 3 1 2 E pag [ f ( E pag , p ) e - yo ( E pag , p ) x + f ( - E pag , p ) e yo ( E pag , - p ) x ] = re 3 pag ( 2 π ) 3 1 2 E pag [ f ( E pag , p ) e - yo ( E pag , p ) x + f ( - E pag , - p ) e yo ( E pag , p ) x ] ϕ ( X ) = re 3 pag ( 2 π ) 3 1 2 mi pag El F ( mi pag , pag ) mi - yo ( mi pag , pag ) X + F ( - mi pag , pag ) mi - yo ( - mi pag , pag ) X ] = re 3 pag ( 2 π ) 3 1 2 mi pag El F ( mi pag , pag ) mi - yo ( mi pag , pag ) X + F ( - mi pag , pag ) mi yo ( mi pag , - pag ) X ] = re 3 pag ( 2 π ) 3 1 2 mi pag El F ( mi pag , pag ) mi - yo ( mi pag , pag ) X + F ( - mi pag , - pag ) mi yo ( mi pag , pag ) X ]
donde hemos intercambiado p - p pag - pag En el segundo término. Identificamos la expansión habitual (foto de Heisenberg).
ϕ ( x ) = re 3 pag ( 2 π ) 3 1 2 E pag - - - - [ a pag mi - i p x + b pag mi i p x ] ϕ ( X ) = re 3 pag ( 2 π ) 3 1 2 mi pag El una pag mi - yo pag X + segundo pag mi yo pag X ]
con
una pag segundo pag = 1 2 E pag - - - - F ( E pag , p ) = 1 2 E pag - - - - F ( - E pag , - p ) una pag = 1 2 mi pag F ( mi pag , pag ) segundo pag = 1 2 mi pag F ( - mi pag , - pag )
( F ( E pag , p ) F ( mi pag , pag ) y F ( - E pag , - p ) F ( - mi pag , - pag ) son los operadores de aniquilación y creación relativisticamente normalizados ). Si ϕ ϕ Es real, sabemos por el análisis de Fourier que ϕ ^ ( p ) = ϕ ^ ( - p ) ϕ ^ ( pag ) = ϕ ^ ( - pag ) , que se traduce inmediatamente en una pag = b pag una pag = segundo pag .

Hasta ahora, el campo es completamente clásico ( una pag una pag y segundo pag segundo pag son simplemente números complejos, y es conjugación compleja). Así, incluso el campo clásico tiene dos tipos de excitaciones: soluciones de frecuencia positiva (con coeficientes una pag una pag ) y soluciones de frecuencia negativa (con coeficientes segundo pag segundo pag ). Tras su cuantificación, corresponden a partículas y antipartículas.

¡La razón fundamental por la que necesitamos dos especies diferentes de operadores de creación / aniquilación para un campo escalar complejo es la relatividad!

Una teoría relativista, como la teoría cuántica de campos, debe ser causal. Esto implica que la amplitud de una partícula a ser creada en X X y aniquilado en y y , con x - y X - y Espacio como, debe desaparecer. La partícula no puede propagarse fuera del cono de luz. Suponer que

ϕ ( x ) = re 3 pag ˜ una pag mi - i p x , (1) (1) ϕ ( X ) = re 3 pag ~ una pag mi - yo pag X ,
dónde re 3 pag ˜ re 3 pag ~ denota la medida invariante. Luego, la amplitud para una partícula que se crea en X X , propagándose y siendo aniquilado en y y instantáneamente es
⟨0 | ϕ ( y , t ) ϕ ( x , t ) | 0⟩ = 0. 0 | ϕ ( y , t ) ϕ ( X , t ) | 0 = 0.
Ya que ϕ ( y , t ) | 0⟩ = 0 ϕ ( y , t ) | 0 = 0 , entonces la condición anterior implica en
⟨0 | [ ϕ ( y , t ) , ϕ ( x , t ) ] | 0⟩ = 0. (2) (2) 0 | El ϕ ( y , t ) , ϕ ( X , t ) ] | 0 = 0.
Por otro lado, conectando (1) en (2) y asumiendo [ a pag un q ] δ ( p - q ) El una pag , una q ] δ ( pag - q ) obtenemos
⟨0 | [ ϕ ( y , t ) , ϕ ( x , t ) ] | 0⟩ 0. 0 | El ϕ ( y , t ) , ϕ ( X , t ) ] | 0 0.

Para obtener ese [ ϕ ( x ) , ϕ ( y ) ] El ϕ ( X ) , ϕ ( y ) ] desaparece fuera del cono de luz pero no dentro, debemos permitir ondas planas de energía / frecuencia negativa en la expansión de campo. El último paso es interpretar estas ondas planas de energía negativa como correspondientes a antipartículas (energía positiva) con momentos opuestos a las partículas correspondientes. Necesitamos por lo tanto dos especies de operadores de creación / aniquilación, una pag un pag una pag , una pag para partículas y segundo pag , b pag segundo pag , segundo pag Para las antipartículas. Uno puede comprobar que

ϕ ( x ) = re 3 pag ˜ ( un pag mi - i p x + b pag mi i p x ) ϕ ( X ) = re 3 pag ~ ( una pag mi - yo pag X + segundo pag mi yo pag X ) ,
satisface
[ ϕ ( x ) , ϕ ( y ) ] = i Δ ( x , y ) El ϕ ( X ) , ϕ ( y ) ] = yo Δ ( X , y ) ,
dónde Δ ( x , y ) = 0 Δ ( X , y ) = 0 para espacios como intervalos y Δ ( x , y ) 0 Δ ( X , y ) 0 de otra manera.